Intégrale de Dirichlet

Bonsoir,
En me baladant sur le net pour chercher une résolution d'exercice je suis tombé sur le texte ci-dessous.
Ma question est la suivante. Comment le (2n+1) disparaît de la borne supérieur de l'intégrale ?
Cordialement,
Matthieu59642

Réponses

  • Bonjour,

    Cette suite de calculs ne mène pas au résultat. Il est écrit $\displaystyle \lim_{n \to +\infty} \int_{0}^{{\pi \over 2}} {\sin((2n+1)u)\over u}du = {\pi \over 2}.$ Ah bon ? Si on utilise cette relation sans démonstration autant écrire tout de go : on sait que $\displaystyle \int_{0}^{+\infty} {\sin x \over x}dx = {\pi \over 2}.$
  • Bonsoir,

    La limite égale à pi/2 est démontré dans les questions précédentes, c'est plutôt le fait que le (2n+1) disparaisse de la borne supérieur, qui me dérange...

    cordialement,

    Matthieu
  • Bonjour,

    Ah bon... C'est le changement de variable : quand on pose $t = (2n+1) u$ alors la borne en $t$, par exemple $A$, devient $A/(2n+1)$ en $u$, non ?
  • Mais oui ! je suis bête, ça faisait longtemps que je n'avais pas fait de changement de variable.
    Merci beaucoup !
    Bonne soirée
    cordialement,
    Matthieu
  • Et si on jouait à "encore une preuve de..." avec cette intégrale? :-)

    Je vous livre une preuve que je ne connaissais pas:

    Soit $x>0$ un réel.


    $\begin{align}
    \int_0^{\tfrac{\pi}{2}} \text{e}^{-x\sin \theta}\cos(x\cos \theta)d\theta&=\int_0^{\tfrac{\pi}{2}} \Re\left(\text{e}^{ix\text{e}^{i\theta}}\right)d\theta\\
    &=\int_0^{\tfrac{\pi}{2}}\left(1-x\sin \theta-\dfrac{x^2}{2!}\cos(2\theta)+\dfrac{x^3}{3!}\sin(3\theta)+...\right)d\theta\\
    &=\dfrac{\pi}{2}-x+\dfrac{x^3}{3\times 3!}-\dfrac{x^5}{5\times 5!}+...\\
    &=\dfrac{\pi}{2}-\int_0^x \dfrac{\sin \theta}{\theta}d\theta\\
    \end{align}$

    En valeur absolue $\displaystyle \int_0^{\tfrac{\pi}{2}} \text{e}^{-x\sin \theta}\cos(x\cos \theta)d\theta$
    est moindre que $\displaystyle \int_0^{\tfrac{\pi}{2}} \text{e}^{-x\sin \theta}d\theta$

    Or, sur l'intervalle $\left[0;\dfrac{\pi}{2}\right]$, on a $\sin \theta\geq \dfrac{2}{\pi}\theta$

    Donc,

    $\displaystyle \int_0^{\tfrac{\pi}{2}} \text{e}^{-x\sin \theta}d\theta\leq \int_0^{\tfrac{\pi}{2}} \text{e}^{-\tfrac{2x\theta}{\pi}}d\theta=\dfrac{\pi}{2\pi}\left(1-\text{e}^{-x}\right)<\dfrac{\pi}{2x}$

    Quand on fait tendre $x$ vers l'infini, cette intégrale tend vers $0$ et donc,

    $\boxed{\displaystyle \int_0^{\infty} \dfrac{\sin \theta}{\theta}d\theta=\dfrac{\pi}{2}}$

    Source:
    Further remarks on $\displaystyle \int_0^{\infty} \dfrac{\sin x}{x}dx$, G.H Hardy, The mathematical gazette, 1916.
    (l'article passe en revue plusieurs preuves de ce résultat, il fait suite à un précédent article ayant la même teneur)

    (G.H Hardy attribue cette preuve à Schlömilch )
  • @ Fin de partie
    Coïncidence, j'ai posé cet exercice en colle en MP avant-hier, chapitre « Intégrales à paramètre ». Je le pose comme dans le texte ci-joint.
    La possibilité de dériver la fonction $\displaystyle x\mapsto F(x)=\int_{0}^{\frac{\pi }{2}}e^{-xe^{it}}dt$ par la formule de Leibniz $ \displaystyle F^{\prime }(x)=\int_{0}^{\frac{\pi }{2}}\frac{\partial }{\partial x}(e^{-xe^{it}})dt$ est grandement simplifiée par le fait qu'il s'agit d'une intégrale proprement dite, sur un segment, d'une fonction de classe $\mathcal{C}^{1}$, et même $\mathcal{C}^{\infty}$.
    C'est pourquoi on posait cet exercice dans les concours bien avant que les théorèmes de convergence dominée ne fussent au programme.
    La propriété remarquable de la fonction $ (x,t) \mapsto f(x,t)=e^{-xe^{it}}$ est : $\frac{\partial }{\partial t}(e^{-xe^{it}})=ix\frac{\partial }{\partial x}(e^{-xe^{it}})$, ce qui permet un calcul aisé de $F^{\prime }(x)$
    Exercice de bon calibre pour une colle, ni trivial, ni infaisable.
    On arrive ainsi au résultat : $ \displaystyle \int_{0}^{+\infty }\frac{\sin t}{t}=\frac{\pi }{2}$, mais aussi : $ \displaystyle \int_{0}^{+\infty }\frac{e^{-t}-\cos t}{t}dt=0$, moins spectaculaire et moins utile sans doute.
    C'est sans doute le calcul le plus simple de cette intégrale. On pourrait en profiter pour refaire une liste des méthodes pour la calculer, ou de retrouver ces méthodes dans ce forum.
    Bonne journée.
    Fr. Ch.
  • c'est typiquement le genre d'intégrale rapide à faire dans $\mathbb{C}$ ... on considère la partie imaginaire de $\displaystyle{\int \frac{e^{it}}{t}dt}$ sur un contour demi-circulaire dans le demi-plan de Poincaré, en laissant le pôle $t=0$ en dehors ... le résidu étant égal à $1$, alors :
    $$\int_0^{+\infty} \frac{\sin t}{t}dt = \frac{1}{2} \Im \left( i \pi \right) = \frac{\pi}{2}$$
  • « C'est typiquement » ... bof... L'intégration sur un contour est UNE des méthodes. Pour l'instant j'en ai six dans ma besace mais au début du XXe siècle, Hardy en avait plus je pense.
    Cette méthode ne nécessite pas le théorème des résidus, mais seulement le fait que l'intégrale d'une forme différentielle exacte sur un contour est nulle. On la traitait donc en Math. Spé. avant que le programme de 2013 ne supprime, avec beaucoup d'autres choses, les intégrales curvilignes, merci aux réformateurs avec leurs gros ciseaux.
  • l'adverbe 'typiquement' allait bien avec 'rapide à calculer dans $\mathbb{C}$' ... et n'était pas censé être extirpé du contexte de la phrase ...
  • Désolé d'insister mais « rapide à calculer dans $\mathbb C$ », je ne suis pas certain d'être d'accord non plus. Il faut considérer l'intégrale d'une forme différentielle complexe sur un contour constitué de deux quarts de cercles reliés par des segments de droites.
    Il faut trouver les limites des intégrales de notre forme différentielle quand le rayon du petit quart de cercle tendant vers $0$ et celui du grand vers $+\infty$, et sur les segments. Il faut le faire.
    C'est effectivement une jolie démonstration, une parmi d'autres. Qui n'est pas plus courte que la démonstration que nous avons évoquée précédemment.
  • et la, on se demande comment calculer $ \displaystyle \int_0^{+\infty} \; \left( \dfrac{\sin (t) }{t } \right)^2 \mathrm dt $ ?
    A demon  wind propelled me east of the sun
  • @gilles benson
    C'est une blague? car
    $\displaystyle \int_0^{+\infty} \; \left( \dfrac{\sin (t) }{t } \right)^2 \mathrm dt=\displaystyle \int_0^{+\infty} \; \left( \dfrac{\sin (t) }{t } \right) \mathrm dt$
    Le 😄 Farceur


  • donc ce n'est pas une blague...Evidemment savoir n'est pas prouver.
    A demon  wind propelled me east of the sun
  • c'est une simple intégration par parties faite de tête, non?
    Le 😄 Farceur


  • Une autre méthode décrite par G.H Hardy,

    En introduisant la fonction,

    $\displaystyle u(a)=\int_0^{+\infty} \dfrac{a\cos (cx)}{a^2+x^2}dx$

    Cette fonction vérifie une équation différentielle qu'on peut résoudre apparemment.
  • Une autre méthode c'est l'utilisation des transformées de Laplace ( La specialité de Jean Lismonde) la TL de $\frac{sint } {t }$ en $s$ est $\frac \pi 2-acrtan(s)$ ensuite il suffit de remplacer $s$ par 0
    Le 😄 Farceur


  • Une autre preuve que je reproduis telle qu'imprimée:

    $\begin{align}
    \displaystyle \int_0^{\infty} \dfrac{\sin x}{x}dx&=\int_0^{\infty} \lim_{a\rightarrow 0} \left(e^{-ax}\dfrac{\sin x}{x}\right)dx\\
    &= \lim_{a\rightarrow 0}\int_0^{\infty} e^{-ax}\dfrac{\sin x}{x}dx\\
    &= \lim_{a\rightarrow 0}\int_0^{\infty} \left(e^{-ax} \int_0^1 \cos (tx)dt\right)dx \\
    &= \lim_{a\rightarrow 0}\int_0^{1}\left(\int_0^{\infty} e^{-ax}\cos (tx)dx\right)dt\\
    &=\lim_{a\rightarrow 0} \int_0^1 \dfrac{a}{a^2+t^2}dt\\
    &=\lim_{a\rightarrow 0} \arctan\left(\dfrac{1}{a}\right)\\
    &=\dfrac{\pi}{2}
    \end{align}$

    PS:
    C'est dans la même veine que la preuve que décrit Gebrane.
  • Encore une autre,

    \begin{align}

    \displaystyle \int_0^{\infty} \dfrac{\sin x}{x}dx&=\dfrac{1}{2}\int_{-\infty}^{\infty} \dfrac{\sin x}{x}dx\\
    &=\dfrac{1}{2} \sum_{k=-\infty}^{+\infty} \int_{k\pi}^{(k+1)\pi}\dfrac{\sin x}{x} dx\\
    &=\dfrac{1}{2} \sum_{k=-\infty}^{+\infty} (-1)^k\int_0^{\pi} \dfrac{\sin x}{x-k\pi}dx\\
    &=\dfrac{1}{2}\int_0^{\pi} \left(\sin x \sum_{k=-\infty}^{+\infty} \dfrac{(-1)^k}{x+k\pi}\right)dx\\
    &=\dfrac{1}{2}\int_0^{\pi} \sin x \text{ cosec } xdx\\
    &=\dfrac{\pi}{2}
    \end{align}
  • Il y a aussi celle-là,

    $\Re(a)>0$, $Re(b)>0$,


    $\displaystyle \int_0^{+\infty}\dfrac{e^{-ax}-e^{-bx}}{x}dx=\int_0^{\infty} \left(\dfrac{1}{a+x}-\dfrac{1}{b+x}\right)dx$

    Je ne sais pas comment il établit cette égalité entre nombres complexes.

    Si on suppose $a,b$ réels $>0$ d'après le théorème de Frullani,

    $\displaystyle \int_0^{+\infty} \dfrac{e^{-ax}-e^{-bx}}{x}dx=\ln(b)-\ln(a)$

    J'imagine que cette égalité continue d'être vraie pour $a,b$ complexes avec $\Re(a)>0$, $\Re(b)>0$,

    et il y a plus qu'à prendre $a=1,b=i$ pour terminer le calcul.
  • Bien vue, cette extension de Cauchy-Frullani au cas complexe.
    Quelle est la référence ? Toujours Hardy ?
    Pour la justification j'ai trouvé ça :
    http://math.stackexchange.com/questions/1807410/frullani-s-theorem-in-a-complex-context

    Mais pour la fin, la partie réelle de $i$ n'est pas $>0$. Encore un passage à la limite à justifier.
  • Hardy donne seulement la première égalité qui évite sans doute le recours à un logarithme de nombre complexe.
    Il écrit, si je comprends bien, qu'il a trouvé cette preuve dans un bouquin d'un certain Bromwich avec lequel il a signé des articles si je lis toujours bien.

    Et la forme de la première intégrale suggère l'utilisation du théorème de Frullani mais qui, à priori, ne concerne que des paramètres $a,b$ réels. Je n'ai pas de doute sur le fait qu'on puisse étendre cette égalité par prolongement analytique.

    PS:
    Merci pour le lien. On y trouve l'explication de l'égalité de Hardy.

    Je recopie cette preuve, en l'adaptant à la situation:

    $\begin{align}
    \int_0^{+\infty}\dfrac{\text{e}^{-ax}-\text{e}^{-bx}}{x}dx&=\int_0^{+\infty}\left(\left(\text{e}^{-ax}-\text{e}^{-bx}\right)\int_0^{+\infty}\text{e}^{-xt}dt\right)dx\\
    &=\int_0^{+\infty}\int_0^{+\infty} \left(\text{e}^{-(a+t)x}-\text{e}^{-(b+t)x}\right) dtdx\\
    &=\int_0^{+\infty} \left(\dfrac{1}{a+t}-\dfrac{1}{b+t}\right)dt
    \end{align}$

    Et cette égalité est vraie pour $\Re(a),\Re(b)>0$ ce qui garantie la convergence de la toute première intégrale sauf erreur.

    PS:
    Pour la suite, il faudrait que cette égalité soit vraie pour $\Re(a),\Re(b)\geq 0$
  • Par ailleurs,

    $\begin{align}
    \int_0^{+\infty} \left(\dfrac{1}{t+1}-\dfrac{1}{t+i}\right)dt&=\int_0^{+\infty} \dfrac{(i-1)(t-i)}{(t+1)(t^2+1)}dt\\
    &=\left [\left( i-1\right) \cdot \left( \frac{\left( 1+i\right) \cdot \mathrm{log}\left( {{t}^{2}}+1\right) }{4}-\frac{\left( 1+i\right) \cdot \mathrm{log}\left( t+1\right) }{2}+\frac{\left( 1-i\right) \cdot \mathrm{arctan}\left( t\right) }{2}\right)\right]_0^{+\infty}\\
    &=i\dfrac{\pi}{2}
    \end{align}$
  • L'extension au cas complexe de l'intégrale de Frullani peut se démontrer en restant dans le cadre du programme de Math-Spé.
    On introduit $F(x)=\displaystyle\int_0^{+\infty}\dfrac{\text{e}^{-t}-\text{e}^{-(1+ix)t}}tdt$.
    Par dérivation sous le signe intégral on obtient:
    $F(x)=\dfrac12\ln(1+x^2)+i\arctan x$.

    On en déduit, en utilisant l'intégrale de Frullani dans le cas réel, pour $a>0$ et $c>0$:

    $\displaystyle\int_0^{+\infty}\dfrac{\text{e}^{-(c+id)t}-\text{e}^{-(a+ib)t}}tdt=\dfrac12\ln\dfrac{a^2+b^2}{c^2+d^2}+i\arctan\dfrac ba-i\arctan\dfrac dc$ qui est bien égal à $\ln(a+ib)-\ln(c+id)$.
  • S'il vous plaît n'oubliez pas Cauchy dans la dénomination de cette intégrale de Cauchy-Frullani.
  • Bonjour

    Gebrane a l'amabilité de me citer à propos de la méthode des transformées de Laplace

    je réponds à son appel avec la transformée : $$\int_0^{+\infty}e^{-tx}\frac{sin(at)}{t}dt = Arctan\frac{a}{x}$$

    avec x variable réelle positive et a paramètre différent de 0

    si a = 1 et lorsque x tend vers 0+ il vient l'intégrale de Dirichlet : $$\int_0^{+\infty}\frac{sint}{t}dt = \frac{\pi}{2}$$

    cordialement
  • bonsoir

    la détermination de l'intégrale de Dirichlet peut se faire
    avec les applications intégrales de la fonction Gamma en particulier pour x > - 2 :
    $$\int_0^{+\infty}t^x.sint.dt=sin\frac{\pi}{2}(x+1).\Gamma(x+1)$$

    lorsque x tend vers - 1 alors $sin\frac{\pi}{2}(x+1) \sim \frac{\pi}{2}(x+1)$

    or $(x+1)\Gamma(x+1) = \Gamma(x+2)$ qui tend vers 1 donc le résultat de l'intégration est bien $\frac{\pi}{2}$

    je signale que pour $x = - \frac{1}{2}$ on retrouve l'intégrale de Fresnel soit : $$\int_0^{+\infty}\frac{sint}{\sqrt{t}}dt = \sqrt{\frac{\pi}{2}}$$

    cordialement
Connectez-vous ou Inscrivez-vous pour répondre.